You are on page 1of 19

Javier v.

Veridiano II
FACTS:
Javier filed a Miscellaneous Sales Application for lot 1641. She later
instituted a complaint for forcible entry against Babol, alleging that she was
forcibly dispossessed of a portion of said land.
The case for forcibly entry was however dismissed as it was found by the
court that the occupied portion was outside Lot 1641.
The same was dismissed on appeal.
Javier was eventually granted a Miscellaneous Sales Patent and issued an
OCT for lot 1641. Babol, however had sold the property he was occupying,
including a portion of 200 square meters to Rosete.
Javier demanded the surrender of the same area from Rosete who
repeatedly refused to comply.
After 4 years, Javier instituted a complaint for quieting of title and recovery
of possession with damages against Babol and Rosete. Rosete moved to
dismiss the complaint on the ground of res judicata. The CFI sustained the
argument of Rosete and granted his motion to dismiss.
Javier contends that res judicata cannot apply in the instant case since
there is no identity of parties and causes of action between her complaint
for forcible entry, which had long become final and executory, and her
subsequent petition for quieting of title.
Javier maintains that there is no identity of causes of action since the first
case was for forcible entry, which is merely concerned with the possession
of the property, whereas the subsequent case was for quieting of title,
which looks into the ownership of the disputed land.
ISSUE: Whether or not there are really different causes of action between the
forcible entry case and the later quieting of title case.
HELD: Yes.
For res judicata to bar the institution of a subsequent action the following
requisites must concur:
1. There must be a final judgment or order;
2. The court rendering the judgment must have jurisdiction over the
subject matter; (3) The former judgment is a judgment on the merits;
and,
3. There is between the first and second actions identity of (4a) parties,
(4b) of subject matter and (4c) of causes of action.
Javier's argument that there is no identity of parties between the two
actions is without merit.
We have repeatedly ruled that for res judicata to apply, what is required is
not absolute but only substantial identity of parties. But, there is merit in
Javier's argument that there is no identity of causes of action.
"The only issue in an action for forcible entry is the physical or material
possession of real property, that is, possession de facto and not possession
de jure.
The philosophy underlying this remedy is that irrespective of the actual
condition of the title to the property, the party in peaceable quiet
possession shall not be turned out by strong hand, violence or terror."
A judgment rendered in a case for recovery of possession is conclusive only
on the question of possession and not on the ownership.
It does not in any way bind the title or affects the ownership of the land or
building.
On the other hand, Civil Case No. 2203-0 is in reality an action to recover a
parcel of land or an accion reivindicatoria under Art. 434 of the Civil Code,
and should be distinguished from Civil Case No. 926, which is an accion
interdictal.
Accion interdictal, which is the summary action for forcible entry
(detentacion) where the defendant's possession of the property is illegal ab
initio, or the summary action for unlawful detainer (desahuico) where the
defendant's possession was originally lawful but ceased to be so by the
expiration of his right to possess, both of which must be brought within one
year from the date of actual entry on the land, in case of forcible entry, and
from the date of last demand, in case of unlawful detainer, in the proper
municipal trial court or metropolitan trial court; accion publiciana which is a
plenary action for recovery of the right to possess and which should be
brought in the proper regional trial court when the dispossession has lasted
for more than one year; and, accion reivindicatoria or accion de
reivindicacion which seeks the recovery of ownership and includes the jus
utendi and the jus fruendi brought in the proper regional trial court.
Accion reivindicatoria or accion de reivindicacion is thus an action whereby
plaintiff alleges ownership over a parcel of land and seeks recovery of its
full possession.
It is different from accion interdictal or accion publiciana where plaintiff
merely alleges proof of a better right to possess without claim of title.
In Civil Case No. 926 Javier merely claimed a better right or prior
possession over the disputed area without asserting title thereto.
It should be distinguished from Civil Case No. 2203-0 where she expressly
alleged ownership.
German Management & Services vs. Court of Appeals
Facts:
Spouses Cynthia Cuyegkeng Jose and Manuel Rene Jose, residents of
Pennsylvania, Philadelphia, USA are the owners of a parcel of land situated
in Sitio Inarawan, San Isidro, Antipolo, Rizal, with an area of 232,942 sq. m.
(TCT 50023 of the Register of Deeds Rizal issued 11 September 1980
cancelling TCT 56762/ T-560).
The land was originally registered on 5 August 1948 in the Office of the
Register of Deeds Rizal as OCT 19, pursuant to a Homestead Patent granted
by the President of the Philippines on 27 July 1948, under Act 141.
On 26 February 1982, the spouses Jose executed a special power of
attorney authorizing German Management Services to develop their
property into a residential subdivision.
Consequently, on 9 February 1983 the German Management obtained
Development Permit 00424 from the Human Settlements Regulatory
Commission for said development. Finding that part of the property was
occupied by Gernale and Villeza and 20 other persons, German
Management advised the occupants to vacate the premises but the latter
refused.
Nevertheless, German Management proceeded with the development of the
subject property which included the portions occupied and cultivated by
Gernale, et.al.
Gernale, et.al. filed an action for forcible entry against German Management
before the MTC Antipolo, Rizal, alleging that they are mountainside farmers
of Sitio Inarawan who have occupied and tilled their farmholdings some 12
to 15 years prior to the promulgation of PD27, and that they were deprived
of their property without due process of law when German Management
forcibly removed and destroyed the barbed wire fence enclosing their
farmholdings without notice and bulldozing the rice, corn, fruit bearing trees
and other crops that they planted by means of force, violence and
intimidation.
On 7 January 1985, the MTC dismissed Gernale et.al.s complaint for
forcible entry. On appeal, the RTC Antipolo, Rizal, Branch LXXI sustained
the dismissal by the MTC.
Gernale then filed a petition for review with the Court of Appeals. On 24
July 1986, said court gave due course to their petition and reversed the
decisions of the MTC and the RTC.
The Appellate Court held that since Gernale, et.al. were in actual possession
of the property at the time they were forcibly ejected by German
Management, they have a right to commence an action for forcible entry
regardless of the legality or illegality of possession.
German Management moved to reconsider but the same was denied by the
Appellate Court in its resolution dated 26 September 1986.
Hence the present recourse.
Issues:
1. Whether the mountainside farmers are allowed by law to commence an
action for forcible entry.
2. Whether German Management & Services may validly invoke the doctrine of
self-help.
Held:
1. Yes.
Notwithstanding the claim that German Management was duly authorized
by the owners to develop the subject property, the actual possessors can
commence a forcible entry case against the former because ownership is
not in issue.
Forcible entry is merely a quieting process and never determines the actual
title to an estate.
Title is not involved.
Gernale et.al were already in peaceable possession of the property at the
time German Management entered the property, manifested by the fact
that they even planted rice, corn and fruit bearing trees 12 to 15 years prior
to German Managements act of destroying their crops.
Moreover, a prior possessor has security to remain in property until lawfully
ejected by person having better right by accion publiciana or accion
reivindicatoria.
A party may validly claim ownership based on the muniments of title it may
present, such evidence does not responsively address the issue of prior
actual possession raised in a forcible entry case. It must be stated that
regardless of the actual condition of the title to the property, the party in
peaceable quiet possession shall not be turned out by a strong hand,
violence or terror.
Thus, a party who can prove prior possession can recover such possession
even against the owner himself. Whatever may be the character of his prior
possession, if he has in his favor priority in time, he has the security that
entitles him to remain on the property until he is lawfully ejected by a
person having a better right by accion publiciana or accion reivindicatoria.
2. No.
The justification given by petitioner that the drastic action of bulldozing and
destroying the crops of the prior possessor on the basis of the doctrine of
self help (enunciated in Article 429 NCC) is unavailing, because such
doctrine can only be exercised at the time of actual or threatened
dispossession, which is absent in the present case.
When possession has already been lost, the owner must resort to judicial
process for the recovery of property.
This is clear from Article 536 NCC that , in no case may possession be
acquired through force or intimidation as long as there is a possessor who
objects thereto.
He who believes that he has an action or right to deprive another of the
holding of a thing, must invoke the aid of the competent court, if the holder
should refuse to deliver the thing.
Doctrine: A prior possessor has security to remain in property until lawfully
ejected by person having better right by accion publiciana or accion
reivindicatoria. A party may validly claim ownership based on the
muniments of title it may present, such evidence does not responsively
address the issue of prior actual possession raised in a forcible entry case.
It must be stated that regardless of the actual condition of the title to the
property, the party in peaceable quiet possession shall not be turned out by
a strong hand, violence or terror. When possession has already been lost,
the owner must resort to judicial process for the recovery of property.
Felix Caisip vs. People of the Philippines
Facts:
The complainant Gloria Cabalag is the wife of Marcelino Guevarra who
cultivated a parcel of land known as Lot 105-A of Hacienda Palico situated
in sitio Bote-bote, barrio Tampisao, Nasugbu, Batangas.
The said parcel of land used to be tenanted by the deceased father of the
complainant. Hacienda Palico is owned by Roxas y Cia. and administered by
Antonio Chuidian.
The overseer of the said hacienda is Felix Caisip, one of the accused herein.
On May 17, 1958 Roxas y Cia filed a forcible entry case against Guevarra.
The court decided in favour of Roxas y Cia and issued a writ of execution.
The return of the writ showed that possession of Lot 105-A was turned over
to the owner thru Caisip and that Guevarra and Cabalag were given 20 days
from June 6, 1959 to vacate the premises.
It also appears in the record that due to the tenacious attitude of Cabalag,
Caisip sought the help of policemen Federico Villadelrey and Ignacio
Rojales.
On June 17, 1959, Cabalag was seen weeding the portion of Lot 105-A
which was a ricefield.
Caisip approached her and bade her to leave but Cabalag refused to do so
claiming that she and her husband has a right over the property.
She having stuck to this attitude, even when he threatened to call the
police, Caisip went to his co-defendants, Sgt. Rojales and Cpl. Villadelrey,
both of the local police, who were some distance away, and brought them
with him. Rojales told Gloria, who was then in a squatting position, to stop
weeding.
As Gloria insisted on her right to stay in said lot, Rojales grabbed her right
hand and, twisting the same, wrested therefrom the trowel she was
holding.
Thereupon, Villadelrey held her left hand and, together with Rojales,
forcibly dragged her. The appellants maintain that the Court of Appeals
erred in not finding that their acts are justified under Article 429 of the Civil
Code.
Issue: Whether Article 429 of the Civil Code applies in the present case.
Held: NO
Article 429 is inapplicable, Cabalag was given 20 days from June 6, 1959
within which to vacate the premises. Cabalag did not, on June 17, 1959
or within said period invade or usurp said lot.
She had merely remained in possession thereof, even though the hacienda
owner may have become its co-possessor.
Appellants did not repel or prevent in actual or threatened . . . physical
invasion or usurpation.
They expelled Gloria from a property of which she and her husband were in
possession.
It is urged, that, by weeding and refusing to leave Lot 105-A, Gloria had
committed a crime in the presence of the policemen, despite the
aforementioned 20-day period, which, appellants claim, the sheriff had no
authority to grant.
This contention is manifestly untenable, because:
a. said period was granted in the presence of the hacienda owners
representative, appellant Caisip, who, by not objecting thereto,
had impliedly consented to or ratified the act performed by the
sheriff;
b. Gloria and her husband were thereby allowed to remain, and had,
in fact, remained, in possession of the premises, perhaps together
with the owner of the hacienda or his representative, Caisip;
c. the act of removing weeds from the ricefield was beneficial to its
owner and to whomsoever the crops belonged, and, even if they
had not authorized it, does not constitute a criminal offense; and
d. although Gloria and her husband had been sentenced to vacate
the land, the judgment against them did not necessarily imply
that they, as the parties who had tilled it and planted thereon,
had no rights, of any kind whatsoever, in or to the standing
crops, inasmuch as necessary expenses shall be refunded to
every possessor, and the cost of cultivation, production and
upkeep has been held to partake of the nature of necessary
expenses.
It is, accordingly, clear that appellants herein had, by means of violence,
and without legal authority therefor, prevented the complainant from doing
something not prohibited by law, (weeding and being in Lot 105-A), and
compelled her to do something against her will (stopping the weeding and
leaving said lot), whether it be right or wrong, thereby taking the law into
their hands, in violation of Art. 286 of the Revised Penal Code.
Custodio v. Court of Appeals
FACTS:
Pacifico Mabasa owns a parcel of land with a two-door apartment.
Said property may be described to be surrounded by other immovables
owned by petitioner Spouses Custodio, Spouses Santos and Rosalina
Morato.
From the main street P. Burgos, there are two possible passageways to
Mabasas property. One of the tenants of the apartment vacated because
an adobe fence was constructed thereby making the first passageway
narrower in width.
Ma. Cristina Santos testified that she constructed said fence for security
reasons. Morato also constructed her fence and even extended it in such a
way that the entire passageway was enclosed.
It was then that the remaining tenants of the apartment left.
Thereafter, Mabasa filed a case for the grant of an easement of right of way
against petitioners.
The RTC granted the easement of right of way sought by private
respondent.
On appeal, the CA affirmed the decision of the RTC and furthermore,
ordering petitioners to pay private respondent a sum of money for
damages.
ISSUE: Whether the award of damages to private respondent is proper.
HELD: No,
the act of petitioners in constructing a fence within their lot is a valid
exercise of their right as owners.
Article 430 of the Civil Code provides that every owner may enclose or
fence his land or tenements by means of walls, ditches, live or dead
hedges, or by any other means without detriment to servitudes constituted
thereon.
The proper exercise of a lawful right cannot constitute a legal wrong for
which an action will lie, although the act may result in damage to another
The courts can give no redress for hardship to an individual resulting from
action reasonably calculated to achieve a lawful end by lawful means.
There is a material distinction between damages and injury.
Injury is the illegal invasion of a legal right; damage is the loss, hurt, or
harm which results from the injury; and damages are the recompense or
compensation awarded for the damage suffered.
Thus, there can be damage without injury in those instances in which the
loss or harm was not the result of a violation of a legal duty.
In this case, it is true that Mabasa may have incurred losses (damage)
when his tenants left because of the fence made by the Santoses.
However, when Santos built the fence, he was well within his right.
He built the fence inside his property.
There was no existing easement agreement, either by contract or by
operation of law, on his property.
Hence, Santos has all the right to build the fence.
It was only after the judgment in the trial court that the easement was
created which was even conditioned on the payment of Mabasa of the just
compensation.
Santos did not commit a legal injury against Mabasa when he built the
fence, therefore, there is no actionable wrong as basis for the award of
damages.
In this case, the damage has to be borne by Mabasa
Andamo vs. Intermediate Appellate Court
Facts:
Petitioner spouses Andamo owned a parcel of land situated in Biga Silang,
Cavite which is adjacent to that of private respondent corporation,
Missionaries of Our lady of La Salette, Inc.
Within the land of the latter, waterpaths and contrivances, including an
artificial lake, were constructed, which allegedly inundated and eroded
petitioners land, caused a young man to drown, damagaed petitioners
crops and plants, washed away costly fences, endangered the
livesofthepetitioners and their laborers and some other destructions.
This prompted petitioner spouses to file a criminal action for destruction by
means of inundation under Article 324 of the RPC and a civil action for
damages.
Issue: Whether petitioner spouses Andamo can claim damages for destruction
caused by respondents waterpaths and contrivances on the basis of Articles 2176 and
2177 of the Civil Code on quasi-delicts.
Held: Yes.
A careful examination of the aforequoted complaint shows that the civil
action is one under Articles 2176 and 2177 of the Civil Code on quasi-
delicts.
All the elements of a quasi-delict are present, to wit:
1. damages suffered by the plaintiff,
2. fault or negligence of the defendant, or some other person for whose
acts he must respond; and
3. the connection of cause and effect between the fault or negligence of
the defendant and the damages incurred by the plaintiff. 11
Clearly, from petitioners complaint, the waterpaths and contrivances built
by respondent corporation are alleged to have inundated the land of
petitioners.
There is therefore, an assertion of a causal connection between the act of
building these waterpaths and the damage sustained by petitioners.
Such action if proven constitutes fault or negligence which may be the basis
for the recovery of damages.
It must be stressed that the use of ones property is not without limitations.
Article 431 of the Civil Code provides that the owner of a thing cannot
make use thereof in such a manner as to injure the rights of a third
person. SIC UTERE TUO UT ALIENUM NON LAEDAS.
Moreover, adjoining landowners have mutual and reciprocal duties which
require that each must use his own land in a reasonable manner so as not
to infringe upon the rights and interests of others.
Although we recognize the right of an owner to build structures on his land,
such structures must be so constructed and maintained using all reasonable
care so that they cannot be dangerous to adjoining landowners and can
withstand the usual and expected forces of nature.
If the structures cause injury or damage to an adjoining landowner or a
third person, the latter can claim indemnification for the injury or damage
suffered.
Doctrine: It must be stressed that the use of ones property is not without
limitations. Article 431 of the Civil Code provides that the owner of a thing
cannot make use thereof in such a manner as to injure the rights of a third
person. SIC UTERE TUO UT ALIENUM NON LAEDAS.
NATL POWER CORP V IBRAHIM
FACTS:
Lucman G. Ibrahim instituted an action against the National Power
Corporation (NPC) for recovery of possession of land and damages before
the Regional Trial Court of Lanao del Sur on November 23, 1994.
The complaint stated that in 1978 NPC used stealth and without permission
took possession of the sub-terrain area of the questioned lands and therein
constructed underground tunnels.
However, it was only discovered in 1992, when Omar G. Maruhoms
request to install a motorized deep well was rejected by the Marawi City
Water District.
However, NPC refused to vacate and / or pay damages considering that the
area was prone to volcanic and tectonic activities.
Instead, NPC filed an answer stating that the respondents were never in
possession the sub-terrain portion because they failed to present proof of
ownership and the tunnels were government projects for the benefit of all.
On August 7, 1996, The RTC denied the petition to vacate but ordered the
NPC to pay the fair market value plus damages. On June 8, 2005, the Court
of Appeals affirmed the original decision of the RTC.
ISSUE: Whether or not Ibrahim owns the sub-terrain properties?
HELD:YES.
The Court sustains the finding of the lower courts that the sub-terrain
portion of the property similarly belongs to Ibrahim.
This conclusion is drawn from Article 437 of the Civil Code, which provides:
ART. 437. The owner of a parcel of land is the owner of its surface and of
everything under it, and he can construct thereon any works or make any
plantations and excavations which he may deem proper, without detriment
to servitudes and subject to special laws and ordinances.
He cannot complain of the reasonable requirements of aerial navigation.
Thus, the ownership of the land extends to the surface as well as to the
subsoil under it.
Republic of the Philippines v. Court of Appeals
FACTS:
An application for registration of a parcel of land was filed on February 11,
1965, by Jose de la Rosa on his own behalf and on behalf of his three
children.
The land, situated in Tuding, Itogon, Benguet Province, was divided into 9
lots.
According to the application, Lots 1-5 were sold to Jose de la Rosa and Lots
6-9 to his children by Mamaya Balbalio and Jaime Alberto, respectively, in
1964.
In support of the application, both Balbalio and Alberto testified that they
had acquired the subject land by virtue of prescription Balbalio claimed to
have received Lots 1-5 from her father shortly after the Liberation.
Alberto said he received Lots 6-9 in 1961 from his mother, Bella Alberto.
She was corroborated by Felix Marcos, who recalled the earlier possession
of the land by Alberto's father.
Benguet opposed on the ground that the June Bug mineral claim covering
Lots 1-5 was sold to it on September 22, 1934, by the successors-in-interest
of James Kelly, who located the claim in September 1909 and recorded it on
October 14, 1909.
From the date of its purchase, Benguet had been in actual, continuous and
exclusive possession of the land in concept of owner.
Atok alleged that a portion of Lots 1-5 and all of Lots 6-9 were covered by
the Emma and Fredia mineral claims located by Harrison and Reynolds on
December 25, 1930, and recorded on January 2, 1931, in the office of the
mining recorder of Baguio.
These claims were purchased from these locators on November 2, 1931, by
Atok, which has since then been in open, continuous and exclusive
possession of the said lots.
The Bureau of Forestry Development also interposed its objection, arguing
that the land sought to be registered was covered by the Central Cordillera
Forest Reserve under Proclamation No. 217 dated February 16, 1929.
Moreover, by reason of its nature, it was not subject to alienation under the
Constitutions of 1935 and 1973.
The trial court denied the application, holding that the applicants had failed
to prove their claim of possession and ownership of the land sought to be
registered.
The applicants appealed to the respondent court, which reversed the trial
court and affirmed the surface rights of the de la Rosas over the land while
at the same time reserving the sub-surface rights of Benguet and Atok by
virtue of their mining claims. Both Benguet and Atok appealed to the
Supreme Court, invoking their superior right of ownership.
The Republic filed its own petition for review and reiterated its argument
that neither the private respondents nor the two mining companies had any
valid claim to the land because it was not alienable and registerable.
ISSUE: Whether or not Benguet and Atok have a better right over the property in
question.
HELD: Yes.
It is true that the subject property was considered forest land and included
in the Central Cordillera Forest Reserve, but this did not impair the rights
already vested in Benguet and Atok at that time. T
he perfection of the mining claim converted the property to mineral land
and under the laws then in force removed it from the public domain.
By such act, the locators acquired exclusive rights over the land, against
even the government, without need of any further act such as the purchase
of the land or the obtention of a patent over it.
As the land had become the private property of the locators, they had the
right to transfer the same, as they did, to Benguet and Atok.
It is true, as the Court of Appeals observed, that such private property was
subject to the "vicissitudes of ownership," or even to forfeiture by non-user
or abandonment or, as the private respondents aver, by acquisitive
prescription.
The Court of Appeals justified this by saying there is "no conflict of interest"
between the owners of the surface rights and the owners of the sub-surface
rights.
Under the aforesaid ruling, the land is classified as mineral underneath and
agricultural on the surface, subject to separate claims of title. However, the
rights over the land are indivisible and that the land itself cannot be half
agricultural and half mineral. T
he classification must be categorical; the land must be either completely
mineral or completely agricultural.
In the instant case, as already observed, the land which was originally
classified as forest land ceased to be so and became mineral and
completely mineral once the mining claims were perfected.
As long as mining operations were being undertaken thereon, or
underneath, it did not cease to be so and become agricultural, even if only
partly so, because it was enclosed with a fence and was cultivated by those
who were unlawfully occupying the surface.
This is an application of the Regalian doctrine. If a person is the owner of
agricultural land in which minerals are discovered, his ownership of such
land does not give him the right to extract or utilize the said minerals
without the permission of the State to which such minerals belong.
Benguet and Atok have exclusive rights to the property in question by virtue
of their respective mining claims which they validly acquired before the
Constitution of 1935 prohibited the alienation of all lands of the public
domain except agricultural lands, subject to vested rights existing at the
time of its adoption.
The land was not and could not have been transferred to the private
respondents by virtue of acquisitive prescription, nor could its use be shared
simultaneously by them and the mining companies for agricultural and
mineral purposes.
The decision is set aside and that of the trial court is reinstated.
Bachrach Motor Co., Inc. v. Talisay Silay Milling Co.
FACTS:
On December 22, 1923, the Talisay-Silay Milling Co., Inc., was indebted to
the Philippine National Bank.
To secure the payment of its debt, it succeeded in inducing its planters,
among whom, was Mariano Lacson Ledesma, to mortgage their land to the
creditor bank.
And in order to compensate those planters for the risk they were running
with their property under the mortgage, the aforesaid central, by a
resolution passed on that same date, i.e., December 22, 1923, undertook to
credit the owners of the plantation thus mortgaged every year with a sum
equal to two per centum of the debt secured according to yearly balance,
the payment of the bonus being made at once, or in part from time to time,
as soon as the central became free of its obligations to the aforesaid bank,
and of those contracted by virtue of the contract of supervision, and had
funds which might be so used, or as soon as it obtained from said bank
authority to make such payment.
Bachrach Motor Co., Inc. filed a complaint against the Talisay-Silay Milling
Co., Inc., for the delivery of the amount P13,850 or promissory notes or
other instruments or credit for that sum payable on June 30, 1930, as
bonus in favor of Mariano Lacson Ledesma.
The Philippine National Bank filed a third party claim alleging a preferential
right to receive any amount which Mariano Lacson Ledesma might be
entitled to from the Talisay-Silay Milling Co. as bonus, because that would
be civil fruits of the land mortgaged to said bank by said debtor for the
benefit of the central referred to, and by virtue of a deed of assignment,
and praying that said central be ordered to delivered directly to the
intervening bank said sum on account of the latter's credit against the
aforesaid Mariano Lacson Ledesma.
ISSUE: Whether or not the bonus in question is civil fruits
HELD: No.
The said bonus bears no immediate, but only a remote accidental relation
to the land mentioned, having been granted as compensation for the risk of
having subjected one's land to a lien in favor of the bank, for the benefit of
the entity granting said bonus.
If this bonus be income or civil fruits of anything, it is income arising from
said risk, or, if one chooses, from Mariano Lacson Ledesma's generosity in
facing the danger for the protection of the central, but certainly it is not civil
fruits or income from the mortgaged property.
Hence, the amount of the bonus, according to the resolution of the central
granting it, is not based upon the value, importance or any other
circumstance of the mortgaged property, but upon the total value of the
debt thereby secured, according to the annual balance, which is something
quite distinct from and independent of the property referred to.
Civil Fruits under Article 355 of the Civil Code
o Article 355 of the Civil Code considers three things as civil fruits:
First, the rents of buildings; second, the proceeds from leases of
lands; and, third, the income from perpetual or life annuities, or
other similar sources of revenue.
o According to the context of the law, the phrase u otras analogas
refers only to rents or income, for the adjectives otras and
analogas agree with the noun rentas, as do also the other
adjectives perpetuas and vitalicias.
o The civil fruits the Civil Code understands one of three and only
three things, to wit: the rent of a building, the rent of land, and
certain kinds of income.
Bonus not a civil fruit; not an income of the land
o The amount of the bonus, according to the resolution of the
central granting it, is not based upon the value, importance or
any other circumstance of the mortgaged property, but upon the
total value of the debt thereby secured, according to the annual
balance, which is something quite distinct from and independent
of the property referred to.
o As the bonus is not obtained from the land, it is not civil fruits of
that land. It is neither rent of buildings, proceeds from lease of
lands, or income under Article 355 of the Civil Code.
Pacific Farms Inc. v. Esguerra
FACTS:
On October 1, 1956 to March 2, 1957 the Company sold and delivered
lumber and construction materials to the Insular Farms Inc. which the latter
used in the construction of the si buildings at its compound in Bolinao,
Pangasinan, of the total procurement price of P15,000.00, the sum of
P4,710.18 has not been paid. Consequently, the Company instituted a civil
case to recover the unpaid balance and the court sustained their claim.
The defendant sheriff levied th six buildings.
The Pacific Farms, Inc. filed a suit against the Company and the sheriff
asserting ownership over the levied buildings which it had acquired from the
Insular Farms by virtue of absolute sale executed on March 21, 1958.
Pacific prays that the judicial sale of the six buildings be declared null and
void. The trial court rendered judgment annulling the levy and the
certificate of sale.
However, it denied the plaintiff's claim for actual and exemplary damages
on the ground that it was not "prepared to find there was gross negligence
or bad faith on the part of any defendants".
ISSUE: Whether or not the application by analogy of the rules of accession would
suffice for a just adjudication.
HELD: YES
Article 447 of the Civil Code contemplates a principal and an accessory; the
land being considered the principal, and the plantings, constructions or
works, the accessory.
The owner of the land who in good faith - whether personally or through
another - makes constructions or works thereon, using materials belonging
to somebody else, becomes the owner of the said materials with the
obligation however of paying for their value. On the other hand, the owner
of the materials is entitled to remove them, provided no substantial injury is
caused to the landowner.
Otherwise, he has the right to reimbursement for the value of his materials,
Applying article 447 by analogy, the Court consider the buildings as the
principal and the lumber and construction materials that went into their
construction as the accessory.
Thus the appellee, if it does own the six buildings, must bear the obligation
to pay for the values of the said materials; the appellant which
apparently has no desire to remove the materials, and, even if it were
minded to do so, cannot remove them without necessarily damaging the
buildings has the corresponding right to recover the value of the unpaid
lumber and construction materials.
BERNARDO v BATACLAN
FACTS:
Plaintiff Vicente Bernardo acquired a parcel of land fromPastor Samonte
thru a contract of sale.
Thereafter, Bernardoinstituted a case against said vendor to secure
possession of theland.
Bernardo was able to obtain a favorable decision from thecourt.
The plaintiff found the defendant herein, Catalino Bataclan, inthe said
premises.
It appears that he has been authorized by formerowners, as far back as
1922, to clear the land and makeimprovements thereon.
Thus, plaintiff instituted a case againstBataclan in the Court of First
Instance of Cavite.
this case, plaintiff was declared the owner of the land but the defendant
was held to bea possessor in good faith, entitled to reimbursement in the
total sumof P1,642, for work done and improvements made.
Both partiesappealed the decision. The court thereafter made some
modifications by allowing thedefendant to recover compensation amounting
to P2,212 and byreducing the price at which the plaintiff could require the
defendantto purchase the land in question from P300 down to P200
perhectare.
Plaintiff was likewise given 30 days from the date when thedecision became
final to exercise his option,either to sell the landto the defendant or to buy
the improvements from him.
On January 9, 1934, the plaintiff conveyed to the court his desire"torequire
the defendant to pay him the value of the land at therate of P200 per
hectare or a total price of P18,000 for thewhole tract of land."
The defendant indicated that he was unableto pay the landand, on January
24, 1934, an order was issuedgiving the plaintiff 30 days within which to
pay the defendant thesum of P2,212.Subsequently, on April 24, 1934, the
court below, at the instance of the plaintiff and without objection on the
part of the defendant,ordered the sale of the land in question at public
auction.
The landwas sold on April 5, 1935 to Toribio Teodoro for P8,000.
ISSUE: WON DEFENDANT BATACLAN IS STILL ENTITLED TORECOVER THE COURT
MANDATED COMPENSATION ARISINGFROM THE SALE OF THE PROPERTY TO
TORIBIO
HELD: NO.
Manresa, basing on Art 448 of the NCC, where the planter,builder or sower
has acted in good faith, a conflict of rights arisesbetween the owners and it
becomes necessary to protect the ownerof the improvements without
causing injustice to the owner of theland.
The law provided a just and equitable solution by giving theowner of the
land the option to acquire the improvements afterpayment of the proper
indemnity or to oblige the builder or planterto pay for the land and the
sower to pay the proper rent.
In this case,the plaintiff, as owner of the land, chose to require the
defendant, asowner of the improvements to pay for the land.
The defendant avers thathe is a possessor in good faith andthat the
amount of P2,212 to which he is entitled has not yetbeen paid to him.
Defendant further claims that he has a right toretain the land in accordance
with the provisions of article 453 of theCivil Code. While the said argument
is legally tenable, the samemust perforce be denied because defendant
Bataclan has lost hisright of retention as he failed to pay for the land.
The law, as wehave already said, requires no more than that the owner of
the land should choose between indemnifying the owner of the
improvements or requiring the latter to pay for theland.
ROSALES VS. CASTELLFORT
FACTS:
Spouses-petitioners Rodolfo V. Rosales andLily Rosqueta-Rosales
(petitioners) are the registeredowners of a parcel of land with an area of
approximately 315 square meters, covered byTransfer Certificate of Title
(TCT) No. 36856[4] anddesignated as Lot 17, Block 1 of Subdivision Plan
LRCPsd-55244 situated in Los Baos, Laguna
.On August 16, 1995, petitioners discoveredthat a house was being
constructed on their lot,without their knowledge and consent, by
respondentMiguel Castelltort (Castelltort).
It turned out that respondents Castelltortand his wife Judith had purchased
a lot, Lot 16 of thesame Subdivision Plan, from respondent Lina Lopez-
Villegas (Lina) through her son-attorney-in-fact ReneVillegas (Villegas) but
that after a survey thereof bygeodetic engineer Augusto Rivera, he pointed
to Lot17 as the Lot 16 the Castelltorts purchased.
Negotiations for the settlement of the case thus began,with Villegas offering
a larger lot near petitioners lot inthe same subdivision as a replacement
thereof.
In thealternative, Villegas proposed to pay the purchaseprice of petitioners
lot with legal interest.
Bothproposals were, however, rejected by petitionerswhose counsel, by
letter of August 24, 1995, directedCastelltort to stop the construction of and
demolish hishouse and any other structure he may have builtthereon, and
desist from entering the lot.
Petitioners subsequently filed on September 1, 1995 a complaint for
recovery of possession anddamages with prayer for the issuance of a
restrainingorder and preliminary injunction against spouses-respondents
Miguel and Judith Castelltort before theRTC of Calamba, Laguna, docketed
as Civil Case No.2229-95-C
ISSUE: Under Art 448, who has the right of option?
HELD:
Under the foregoing provision (Art 448), thelandowner can choose between
appropriating thebuilding by paying the proper indemnity or obliging
thebuilder to pay the price of the land, unless its value isconsiderably more
than that of the structures, in whichcase the builder in good faith shall pay
reasonablerent.[34]
If the parties cannot come to terms over theconditions of the lease, the
court must fix the termsthereof.
The choice belongs to the owner of the land,a rule that accords with the
principle of accession, i.e.,that the accessory follows the principal and not
theother way around.
Even as the option lies with thelandowner, the grant to him, nevertheless,
ispreclusive.
The landowner cannot refuse to exerciseeither option and compel instead
the owner of thebuilding to remove it from the land.
The raison detre for this provision has beenenunciated thus:Where the
builder, planter or sower hasacted in good faith, a conflict of rights
arisesbetween the owners, and it becomesnecessary to protect the owner
of theimprovements without causing injustice tothe owner of the land.
In view of theimpracticability of creating a state of forcedco-ownership, the
law has provided a justsolution by giving the owner of the land theoption to
acquire the improvements after payment of the proper indemnity, or to
obligethe builder or planter to pay for the land andthe sower the proper
rent.
He cannot refuseto exercise either option.
It is the owner of the land who is authorized to exercise theoption, because
his right is older, andbecause, by the principle of accession, he isentitled to
the ownership of the accessorything
SALVACION MIRANDA, plaintiff-appellants, vs.ESTEBAN FADULLON and
spouses DIONISIO SEGARRA and CLEMENCIA N. DE SEGARRA, defendants-
appellees.
The present appeal was first taken to the Court of Appeals. Later by resolution of the
said court it was certified to us under section 17, paragraph 6 of the Judiciary Act of
1948, as amended, the said Tribunal being of the opinion that the case involved only
questions of law. The facts as may be gathered from the pleadings filed by the parties
may be briefly stated as follows. In the year 1939 one Lucio Tio was the owner of a
parcel of land, lot 1589-J of the Banilad Estate, Cebu, under Transfer Certificate of
Title No. 10548. On December 29, 1939, a power of attorney in favor of one Esteban
Fadullon executed by Lucio Tio was registered in the land records of Cebu City and
annotated on the same certificate of title. In the year 1946, on the strength of the
said power of attorney Fadullon to make the repurchase within this period, the
Segarras about ten days after the expiration of the period filed a sword petition for
the consolidation of their ownership and registered said petition in the office of the
Register of Deeds on May 15, 1946. Apprised of the sale of his property, Lucio Tio on
June 4, 1946, filed a complaint in the Court of First Instance of Cebu, Civil Case No.
181 to annul the sale. Service of summons was made upon the Segarras on June 10,
1946. After hearing the trial court rendered judgment annulling the sale. The Segarras
appealed to the Court of Appeals under CAG. R. No.6550-R and the said Tribunal
affirmed the appealed decision and further required the Segarras to pay plaintiff the
reasonable rentals on the property from the filing of the action until said property
shall have been returned to plaintiff. Upon the decision becoming final the
corresponding writ of execution was issued directing the Sheriff to put plaintiff Tio in
possession of the lot. It turned out however that during the possession of the
property by the Segarras they had introduced improvements thereon consisting of a
building of three rooms and a storage room, and one artesian well, with tower and
water tank and a cement flooring covering about one-third of the lot which according
to the Segarras cost them P5,300. They then filed a motion with the trial court
claiming that they were possessors in good faith of the lot in question, and that they
had introduced the improvements aforementioned in good faith and asked the court
to order the plaintiff to pay for the said improvements valued at P5,300 or to allow
them to buy the land should the plaintiff decide not to pay for the improvements. On
August 28, 1952, the trial court issued the following order:
The attorney for the plaintiff has been accordingly served with copy of defendant's
motion of July 31, 1952, filed through counsel.
As prayed for, without opposition, the plaintiff is hereby ordered to either pay the
defendant spouses, Dionisio Segarra and Clemencia N. Segarra (possessors in good
faith) the sum of P5,300, value of the building erected on the land in question, or
otherwise allow said defendants to purchase the aforementioned lot.
The plaintiff filed a motion for reconsideration claiming that the Segarras were
possessors and builders in bad faith and so were not entitled to reimbursement for
the value of the improvements; that the reason he (plaintiff) did not file an opposition
to the motion of the defendants asking for reimbursement was that he thought that
the trial court was sufficiently informed and impressed with the bad faith with which
defendants bought the land and introduced improvements thereon and that it would
consequently deny their motion; and in support of his motion for reconsideration
plaintiff quoted portions of the decision of the trial court and the Court of Appeals.
Upon the denial of his motion for reconsideration, he took the present appeal.
After a careful review of the record we agree with the plaintiff-appellant. The trial
court in its decision declaring the sale of the land to the defendants null and void and
commenting on the alleged good faith of defendants in buying the property said the
following:
There are two circumstances which seem to stubbornly belie the professed good faith
on the part of the Segarras in buying this property; namely. the circumstances of the
power-of-attorney appearing on the back of the title as of five or six years previous
and the other circumstances of the comparatively limited period of one month granted
vendor Fadullon to redeem the property. Above all these, is the further circumstance
that the said property had already been mortgaged in favor of the Cebu Mutual
Building and Loan Association by virtue of that power-of-attorney.
While the evidence did not disclose a collusion or conspiracy between Fadullon and
the Segarras, yet, considering the short period of one month within which to redeem
and the surrounding circumstances, the possibility of such collusion lingers.
Obviously there was in this transaction a prevailing intention of railroading the
property into a new ownership as may be proven by the fact that said purchasers filed
a sworn petition for consolidating their ownership barely ten days after the expiration
of thirty days, that is, on April 13, 1946, and registered with the office of Register of
Deeds for Cebu twelve days thereafter, or on May 15, 1946.
The Court of Appeals in its decision affirming that of the trial court said:
The Segarra spouses maintain that they are purchasers in good faith. We will now
examine the record on this point. The alleged power of attorney executed by the late
Lucio Tio in favor of appellant Fadullon was registered in the land record of the
Register of Deeds of Cebu Citly and annotated at the back of Transfer Certificate of
Title No. 10548 on December 29, 1939. On the same date, the deed of mortgage in
favor of the Cebu Mutual Building and Loan Association was annotated in the said
Torrens title (Exhibits 1 and 1-B). This encumbrance alone should have been sufficient
to put the Segarra spouses upon an inquiry as to the authority of Fadullon to sell to
them the same property six years later. For instance, the Segarras could have asked
themselves this question: Did not the mortgage of P400 serve the purpose for which
the power of attorney was executed?
The Segarras did not require Fadullon to produce his power of attorney. While it is
true that said power of attorney is annotated at the back of the Torrens title of Tio, it
was still incumbent upon the Segarras to ascertain the scope and authority of
Fadullon under said power of attorney. Fadullon executed the sale with the right to
repurchase within the extraordinary short period of 30 days. This circumstance, again,
should have placed the Segarras on their guards, knowing, as they did, that they were
dealing with an agent under a power of attorney executed before the war. These
unusual circumstances would seem to engender in our minds the possibility of
collusion between the appellants, to hasten the registration of the title of the Segarras
to the land in dispute . .
. . . the transfer of dominion on the property in question to the Segarras was null and
void and of no effect. The new Certificate of Torrens Title No. 392 on the property
now in the name of the Segarras is hereby ordered cancelled and that a new one
issued in the name of Lucio Tio and his wife Salvacion Miranda; ordering the Segarras
to return the possession of said property to plaintiff;
The defendants Segarras are furthermore required to pay plaintiff the reasonable
rentals on the property from the filing of this action until such time as the said
property shall have been returned to plaintiff . . ."
Although neither the trial court nor the Court of Appeals did expressly say and in so
many words that the defendants-appellees were possessors in bad faith, from a
reading of their decisions particularly those we have just quoted, one can logically
infer that that was the conclusion of the two courts, or to say it more mildly, that the
defendants were not possessors in good faith. Moreover, the very fact that the Court
of Appeals sentenced the defendants to pay rentals is an indication, even proof that
defendants were considered possessors and builders in bad faith, or at least that they
were not possessors and builders in good faith. A builder in good faith may not be
required to pay rentals. He has a right to retain the land on which he has built in good
faith until he is reimbursed the expenses incurred by him. Possibly he might be
required to pay rental only when the owner of the land chooses not to appropriate the
improvement and requires the builder in good faith to pay for the land, but that the
builder is unwilling or unable to buy the land, and then they decide to leave things as
they are and assume the relation of lessor and lessee, and should they disagree as to
the amount of the rental then they can go to the court to fix that amount.
Furthermore, plaintiff-appellant in her brief (page 7) says without denial or refutation
on the part of defendants-appellees that they (defendants) applied for a building
permit to construct the improvements in question on December 4, 1946, and the
permit was granted on January 11, 1947, all this about seven months after they
received the summons on June 10, 1946, meaning to say that the improvements were
introduced long after their alleged good faith as possessors had ended.
In view of the foregoing, the appealed order of August 28, 1952 and the order of
October 15, 1952, denying plaintiff's motion for reconsideration are set aside. With
costs against appellees.

IGNACIO V HILARIO
Facts:
Sometime during the 1940s in Pangasinan, a civil suit arose between
Damian Ignacio and Elias Hilario.
Hilario was the owner of a parcel of land.
He later discovered that Ignacio built some buildings therein (a granary and
a house).
After trial, Judge Antonio Felix of the Court of First Instance of Pangasinan
ruled that both were in good faith (Hilario was the owner in good faith while
Ignacio was the builder in good faith).
Judge Felix then spelled out the rights of the parties to wit:
1. Ignacio can retain possession over the buildings he erected until after
he is paid by Hilario for the value of the buildings he erected;
2. Hilario can choose to buy the said buildings or he can choose to sell
Ignacio his land since the value of his land was only P45.00 while the
value of the buildings erected was P2,000.00.
3. However, Hilario refused to avail of his options. Instead, he filed a
motion in court to have Ignacio be ejected and have them destroy the
buildings he erected. Judge Felipe Natividad (he replaced Judge Felix),
granted Hilarios motion.
ISSUE: Whether or not Hilario, the owner in good faith, may eject a builder in good
faith without choosing either to appropriate the building for himself after payment of
its value or to sell his land to the builder in good faith.
HELD: No.
The owner in good faith has to make a choice. He cannot dispense the
options under the law and then eject the builder in good faith. T
this is because both are in good faith.
But when can the owner in good faith compel the builder in good faith to
remove the building he erected? This is only available if after the owner in
good faith chose to sell his land to the builder in good faith and the latter
fails to pay the value of the land within the agree period. Only then can the
owner in good faith compel the builder in good faith to remove the building
he erected
Right of retention of builder in good faith
The owner of the building erected in good faith on a land owned byanother,
is entitled to retain the possession of the land until he ispaid the value of his
building, under article 453.
Article 453 providesthat Necessary expenses shall be refunded to every
possessor; butonly the possessor in good faith may retain the thing until
suchexpenses are made good to him.
Useful expenses shall be refundedto the possessor in good faith with the
same right of retention, theperson who has defeated him in the possession
having the option of refunding the amount of the expenses or paying the
increase invalue which the thing may have acquired in consequence
thereof."
Option of the landowner to pay for the building or sell hisland to the owner
of the building; Right of remotion onlyavailable if he chose the latter and
the owner of the buildingcannot pay
The owner of the land, upon the other hand, has the option, underarticle
361, either to pay for the building or to sell his land to theowner of the
building.
Article 361 provides that The owner of landon which anything has been
built, sown or planted in good faith, shallhave the right to appropriate as his
own the work, sowing orplanting, after the payment of the indemnity stated
in articles 453and 454, or to oblige the one who built or planted to pay the
price of the land, and the one who sowed, the proper rent.
He cannothowever refuse both to pay for the building and to sell the land
andcompel the owner of the building to remove it from the land where itis
erected.
He is entitled to such remotion only when, after havingchosen to sell his
land, the other party fails to pay for the same.
Order amends judgment substantially and thus null andvoid
The order of Judge Natividad compelling the Ignacios to remove
theirbuildings from the land belonging to Hilario and Dres only becausethe
latter chose neither to pay for such buildings nor to sell the land,is null and
void, for it amends substantially the judgment sought tobe executed and is,
furthermore, offensive to articles 361 and 453 of the Civil Code.
Original decision did not become final as it failed todetermine the value of
the buildings and of the lot; and thetime to which the option may be
exercised
In the decision of Judge Felix, the rights of both parties were welldefined
under articles 361 and 453 of the Civil Code, but it failed todetermine the
value of the buildings and of the lot where they areerected as well as the
periods of time within which the option maybe exercised and payment
should be made, these particulars havingbeen left for determination
apparently after the judgment hasbecome final.
The procedure is erroneous, for after the judgment hasbecome final, no
additions can be made thereto and nothing can bedone therewith except its
execution.
And execution cannot be had,the sheriff being ignorant as to how, for how
much, and within whattime may the option be exercised, and certainty no
authority isvested in him to settle these matters which involve exercise of
judicial discretion.
Thus, the judgment rendered by Judge Felix hasnever become final, it
having left matters to be settled for itscompletion in a subsequent
proceeding, matters which remainedunsettled up to the time the petition is
filed in the present case.
DEPRA V. DUMLAO
FACTS:
Dumlao is the owner of a parcel of land in Iloilo, while Depra owns the lot
adjoining his.
Dumlao built his house on his own land, but the kitchen encroached about
34 sq.m on Depras property.
Upon finding this, Depras mom ordered Dumlao to move back from his
encroachment, then subsequently filed an action for unlawful detainer
against Dumlao.
The lower court found that Dumlao was a builder in good faith, and ordered
him to pay rent (PhP5.00/month) forced lease between the parties.
Depra refused to accept the rentals so Dumlao deposited this with the MTC.
Neither party appealed judgment so this became final and executory.
1 year later, though, Depra filed an complaint for Quieting of Title.
Dumlao contested this, stating that the suit is barred by res judicata.
But Depra averred that the lower court did not have jurisdiction to rule on
encumbrances of real property only the CFI has jurisdiction.
ISSUE:
1. Whether or not the order of forced lease decreed in the unlawful detainer
case is valid.
2. Whether or not the subsequent case of res judicata is barred by prescription
due to the prior case of unlawful detainer.
HELD:
1. No.
The judgment of forced lease is improper. A forced lease, just like co-
ownership is not favored.
It should be considered that the parties themselves stipulated that
Dumlao, the builder, was in good faith and it was later found that Depra,
the owner, was also in good faith.
Hence, what applies is the provisions of Article 448 of the Civil Code,
which provides in sum that:
1. Builder in good faith entitled to retain the possession of the land on
which he built in good faith until he is paid the value of the building he
built in good faith;
2. Owner in good faith has the option to either (i) pay for the building
OR (ii) sell his land to the builder in good faith but builder cannot be
forced to buy said land if the same is considerably more than the value
of the building.
Forced rent only comes in if the owner exercises his right to sell the land
but the builder rejects it by reason of the price thereof being considerably
more than the value of the building in such case, the parties shall agree
to the terms of the lease, if they cant agree then they may bring the issue
to court.
2. No.
The action for quieting of title is not barred by reason of res judicata.
The cause of action in the unlawful detainer case involves possession while
the cause of action in the quieting of title case involves ownership.
Furthermore, the Rules of Court explicitly provides that judgment in a
detainer case shall not bar an action between the same parties respecting
title to the land.
ORTIZ v KAYANAN
Facts:
Homestead Application Lot belonged to Dolorico II, Ortizs ward located in
Barrio Cabuluan, Calauag,Quezon
Dolorico II named as successor and heir his uncle Dolorico, then died.
All this time Ortiz was in possession and cultivation of the property.
Dolorico relinquished rights over property in favour of Comintan and
Zamora.
Court found Ortiz to be in good faith, but held the public bidding to be valid.
If petitioner was not found tobe the winner, Comintan and Zamora are to
reimburse him for P13,632. Ortiz is to retain possession untilthe amount is
paid.
CA affirmed RTC
Respondent Judge discovered that after the decision of the lower courts,
Ortiz collected tolls on portionsof the land even if he had not introduced any
improvements on said portions estimated to amount toP25,000.
Petitioner contends that he is entitled to the fruits of the property while the
P13,632 has yet to be paid,this being considered as civil fruits.
Issue: WON petitioner is entitled to fruits while Comintan and Zamora have yet to
pay the indemnity duepetitioner
HELD: NO.
Before possession is legally interrupted, possessor in good faith is entitled
to fruits.
This right ceasesupon defects being known.
This is known as a right to retention, for the creditor to obtain payment of
adebt.
Also we must consider that tolls were collected from portions with no
improvements of petitioner ,therefore he really has no right to said fruits.
IGNAO VS. IAC
FACTS:
Florencio Ignao and his uncles (private respondents) were co-owners of a
parcel of land. This was originally owned by BaltazarIgnao, who married
twice.
In his first marriage, he had four children,including the father of the
petitioner.
In his second marriage, he alsohad four children who waived their rights
over the controverted land. Justo, Florencios father owned 5/8 of the land.
Thereafter, Justoacquired 1/8 share of brother Leon for P500, which was
later sold tohis son Florencio for the same amount. When Justo died,
Florencioinherited the 5/8 share of his father, which brought his land share
to6/8.
Private respondents Juan and Isidro each has 1/8 share on
theland.Petitioner brought an action for partition. Before it was promulgated
Florencio sol 134 sqm of his share.
The decision for partition allotted2/8 of the land to private respondents.
However, no actual partitionwas effected.
Thus, petitioner instituted a complaint for recovery of possession of real
property against private respondents, because thearea occupied by the two
houses built by private respondentsexceeded the portion allotted to them
Trial court ruled that theprivate respondents are builders in good faith.
ISSUE: Whether or not the provisions of Art 448 should apply on aproperty held in
common
HELD: YES
When the co-ownership is terminated by a partition and it appears that the
house of an erstwhile co-owner has encroached upon a portion pertaining
to another co-owner which was however made in good faith, then the
provisions of Article 448 should apply to determine the respective rights of
the parties.
Florencio Ignao is directed within thirty (30) days from entry of judgment to
exercise his option to either appropriate as his own the portions of the
houses of Juan and Isidro Ignao occupying his land upon payment of
indemnity in accordance with Articles 546 and 548 of the Civil Code, or sell
to private respondents the 101 square meters occupied by them at such
price as may be agreed upon.
Should the value of the land exceed the value of the portions of the houses
that private respondents have erected thereon, private respondents may
choose not to buy the land but they must pay reasonable rent for the use of
the portion of petitioners land as may be agreed upon by the parties.
In case of disagreement, the rate of rental and other terms of the lease
shall be determined by the trial court.
Otherwise, private respondents may remove or demolish at their own
expense the said portions of their houses encroaching upon petitioners
land.
Filipinas Colleges, Inc. v. Garcia Timbang, et. al.,
FACTS:
After appropriate proceedings, the Court of Appeals held, among other
things, that Filipinas Colleges, Inc. are declared to have acquired the rights
of the spouses Timbang in the questioned lots, they are ordered to pay the
spouses Timbang in the amount of P15,807.90 plus such other amount
which said spouses might have paid or had to pay.
On the other hand, Maria Gervacio Blas was also declared to be a builder in
good faith of the school building constructed in the lot in question and was
entitled to be paid the amount of P19,000.00 for the same.
Also, in case that Filipinas Colleges, Inc. failed to deposit the value of the
land, which after liquidation was fixed at P32,859.34, within the 90-day
period set by the Court, Filipinas Colleges would lose all its rights to the
land and the spouses Timbang would then become the owners thereof.
If that is the case, the Timbangs are ordered to make known to the court
their option under Article 448 of the Civil Code whether they would
appropriate the building in question, in which even they would have to pay
Filipinas Colleges, Inc. the sum of P19,000.00, or would compel the latter to
acquire the land and pay the price thereof.
Filipinas Colleges, Inc. failed to pay the sum of P32,859.34 so the spouses
Timbang made known to the court their decision that they had chosen not
to appropriate the building but to compel Filipinas Colleges, Inc., for the
payment of the sum of P32,859,34 which was granted by the Court.
As a consequence of which, a writ of execution was issued. Meanwhile, Blas
filed a motion for execution of her judgment representing the unpaid
portion of the price of the house sold to Filipinas which was granted. Levy
was made on the house in virtue of the writs of execution.
Then, the Sheriff of Manila sold the building in public auction in favor of the
spouses Timbang, as the highest bidders.
Several motion were the subsequently filed before the lower court wherein
the court held that:
a. the Sheriff's certificate of sale covering a school building sold at
public auction was null and void unless within 15 days from notice
of said order spouses Timbang shall pay to Blas the sum of
P5,750.00 that the spouses Timbang had bid for the building at
the Sheriff's sale;
b. that Filipinas is owner of 245.00/32,859.34 undivided interest in
Lot No. 2-a on which the building sold in the auction sale is
situated; and
c. that the undivided interest of the Filipinas in the lot should be
sold to satisfy the unpaid portion of the judgment in favor of Blas
and against Filipinas in the amount of P8,200.00 minus the sum
of P5,750.00.
The spouses Timbang contends that because the builder in good faith has
failed to pay the price of the land after the owners thereof exercised their
option under Article 448 of the Civil Code, the builder lost his right of
retention provided in Article 546 and that by operation of Article 445, the
spouses Timbang as owners of the land automatically became the owners
ipso facto of the school building.
ISSUE: Whether or not the spouses Timbang automatically become the owners of the
building upon failure of Filipinas to pay the value of the land.
HELD: No.
Based on Article 448 and 546 of the New Civil Code, the owner of the land
has the right to choose between appropriating the building by reimbursing
the builder of the value thereof or compelling the builder in good faith to
pay for his land.
Even this second right cannot be exercised if the value of the land is
considerably more than that of the building.
In addition to the right of the builder to be paid the value of his
improvement, Article 546 gives him the corollary right of retention of the
property until he is indemnified by the owner of the land.
There is nothing in the language of these two articles, 448 and 546, which
would justify the conclusion of appellants that, upon the failure of the
builder to pay the value of the land, when such is demanded by the land-
owner, the latter becomes automatically the owner of the improvement
under Article 445.
The case of Batacla vs Bernardo cannot be applied in this case in the sense
that although it is true it was declared therein that in the event of the
failure of the builder to pay the land after the owner thereof has chosen this
alternative, the builder's right of retention provided in Article 546 is lost,
nevertheless there was nothing said that as a consequence thereof, the
builder loses entirely all rights over his own building.
Also, in the present case, the Court of Appeals has already adjudged that
appellee Blas is entitled to the payment of the unpaid balance of the
purchase price of the school building.
Blas is actually a lien on the school building are concerned.
The order of the lower court directing the Timbang spouses, as successful
bidders, to pay in cash the amount of their bid in the sum of P5,750.00 is
therefore correct.
Manotok Realty v. Tecson
FACTS:
Petitioner Manotok Realty filed a complaint against Nilo Madlangawa for
recovery of possession with damages with the Court of First Instance of
Manila.
Said court rendered judgment declaring Madlangawa as a builder-possessor
in good faith; ordering the company to recognize the right of Madlangawa
to remain in Lot 345, Block 1, of the Clara Tambunting Subdivision until
after he shall have been reimbursed by the company the sum of P7,500.00,
without pronouncement as to costs.
Not satisfied with the trial courts decision, petitioner appealed to the Court
of Appeals and upon affirming the trial courts decision, it elevated the case
to the Supreme Court.
On July 13, 1977, the Supreme Court issued a resolution denying Manotoks
petition for lack of merit. Petitioner then filed with the trial court (Judge
Jose H. Tecson), a motion for the approval of the companys exercise of
option and for satisfaction of judgment.
However, Judge Tecson denied the motion for approval.
Hence, this petition is filed.
ISSUE: Whether or not respondent Judge Tecson can deny petitioners (landowner)
motion to avail of its option.
HELD: No.
There is, therefore, no basis for the respondent judge to deny the
petitioners motion to avail of its option to appropriate the improvements
made on its property.
Neither can the judge deny the issuance of a writ of execution because the
private respondent was adjudged a builder in good faith or on the ground of
peculiar circumstances which supervened after the institution of this case,
like, for instance, the introduction of certain major repairs of and other
substantial improvements because the option given by law belongs to the
owner of the land.
Under Article 448 of the Civil Code, the right to appropriate the works or
improvements or to oblige the one who built or planted to pay the proper
price of the land belongs to the owner of the land.
The only right given to the builder in good faith is the right of
reimbursement of necessary expenses for the preservation of the land; the
builder cannot compel the landowner to sell such land to the former.
SPOUSES DEL CAMPO V. ABESIA
Facts:
This case involves a parcel of land, situated at the corner of F. Flores and
Cavan Streets, Cebu City. An action for partition was filed by plaintiffs in the
CFI of Cebu. Plaintiffs and defendants are co-owners pro indiviso of this lot
in the proportion of and 1/3 share each, respectively.
The trial court appointed a commissioner in accordance with the agreement
of the parties. ,the Id commissioner conducted a survey, prepared a sketch
plan and submitted a report to the trial court on May 29, 1976,
recommending that the property be divided into two lots: Lot 1161-A with
an area of 30 square meters for plaintiffs and Lot No. 1161-B with an area
of 15 square meters for the defendants.
The houses of plaintiffs and defendants were surveyed and shown on the
sketch plan.
The house of defendants occupied the portion with an area of 5 square
meters of Lot 1161-A of plaintiffs.
The parties manifested their conformity to the report and asked the trial
court to finally settle and adjudicate who among the parties should take
possession of the 5 square meters of the land in question.

Issue: Whether or Not Article 448 of the Civil Code is applicable to a builder in good
faith when the property involved is owned in common.

Held: YES
When the co-ownership is terminated by the partition and it appears that
the house of defendants overlaps or occupies a portion of 5 square meters
of the land pertaining to plaintiffs which the defendants obviously built in
good faith, then the provisions of Article 448 of the new Civil Code should
apply.
Manresa and Navarro Amandi agree that the said provision of the Civil Code
may apply even when there was co-ownership if good faith has been
established.
Applying the aforesaid provision of the Civil Code, the plaintiffs have the
right to appropriate said portion of the house of defendants upon payment
of indemnity to defendants as provided for in Article 546 of the Civil Code.
Otherwise, the plaintiffs may oblige the defendants to pay the price of the
land occupied by their house.
However, if the price asked for is considerably much more than the value
of the portion of the house of defendants built thereon, then the latter
cannot be obliged to buy the land.
The defendants shall then pay the reasonable rent to the plaintiff upon such
terms and conditions that they may agree.
In case of disagreement, the trial court shall fix the terms thereof. Of
course, defendants may demolish or remove the said portion of their house,
at their own expense, if they so decide.
Article 448 of the New Civil Code provides as follows: Art. 448. The owner
of the land on which anything has been built, sown, or planted in good
faith, shall have the right to appropriate as his own the works, sowing or
planting, after payment of the indemnity provided for in articles 546 and
548, or to oblige the one who built or planted to pay the price of the land,
and the one who sowed, the proper rent.
However, the builder or planter cannot be obliged to buy the land if its
value is considerably more than that of the building or trees.
In such case, he shall pay reasonable rent, if the owner of the land does
not choose to appropriate the building or trees after proper indemnity.
The parties shall agree upon the terms of the lease and in case of
disagreement, the court shall fix the terms thereof
Pecson v. Court of Appeals
FACTS:
Pedro Pecson owned a commercial lot situated in Kamias street, Quezon
City, on which he built a a four-door, two-storey apartment building.
But because of failure to pay realty taxes amounting to P12,000.00, the
commercial lot owned was sold at a public auction.
It was purchased by Nepomuceno, which later sold the same to the Nuguid
spouses for P103,000 on October 12, 1983.
Pecson then challenged the sale, alleging that the apartment building,
contrary to the claim of the Nuguid spouses, was not included in the sale.
The lower court judged in favor of Pecson, declaring that the apartment
building was indeed not included in the subject sale.
The Court of Appeals affirmed the same. The Spouses Nuguid then filed a
motion for delivery of possession of the lot and the apartment building.
The lower court ruled in favor of the private respondents, but subject to the
reimbursement to Pecson of the cost of constructing the apartment building
minus the rents due to the spouses (calculated at P21,000 from June 23,
1993 to September 23, 1993).
With the said decision at hand, the spouses then made a move to eject
Pecson and as well as the tenants residing therein.
However, the spouses have yet to pay Pecson for the construction costs.
ISSUE: Whether the Nuguid Spouses can eject Pecson even if reimbursement hasnt
been given for the construction costs.
HELD: No.
The Court ruled that since the spouses still havent reimbursed Pecson for
the cost of construction of the building, the latter has the right to retain the
property, and along with it, the fruits of which during such possession.
The court ruled that though Article 448 do not apply in the case at bar.
By its clear language, Article 448 refers to a land whose ownership is
claimed by two or more parties, one of whom has built some works, or
sown or planted something.
The building, sowing or planting may have been made in good faith or in
bad faith. As in this case, since the owner himself was the one who
constructed the improvement, good faith and bad faith becomes irrelevant.
However, by analogy, the indemnity may be applied, considering that the
primary intent of Article 448 is to avoid a state of forced co-ownership and
that the parties agree that Articles 448 and 546 of the Civil Code are
applicable and indemnity for the improvements may be paid, although they
differ as to the basis of the indemnity.
Since the spouses have opted to appropriate the apartment building,
Pecson is thus entitled to the possession and enjoyment of the apartment
building, until he is paid the proper indemnity, as well as of the portion of
the lot where the building has been constructed.
This is so because the right to retain the improvements while the
corresponding indemnity is not paid implies the tenancy or possession in
fact of the land on which it is built, planted or sown.
The petitioner not having been so paid, he was entitled to retain ownership
of the building and, necessarily, the income therefrom
SPOUSES JUAN NUGUID AND ERLINDA T.NUGUID VS. CA
FACTS:
Pedro P. Pecson owned a commercial lot onwhich he built a four-door two-
storey apartmentbuilding.
For failure to pay realty taxes, the lot wassold at public auction by the City
Treasurer toMamerto Nepomuceno, who in turn sold it for P103,000 to the
spouses Juan and Erlinda Nuguid.
Pecson challenged the validity of the auctionsale before the RTC of Quezon
City, the RTC upheldthe spouses title but declared that the four-door two-
storey apartment building was not included in theauction sale.
This was affirmed by the CA and by theSC.The Nuguids became the
uncontestedowners of commercial lot.
The Nuguid spousesmoved for delivery of possession of the lot and
theapartment building.
ISSUE: WON the Nuguids should reimburse Pecson for thebenefits derived from the
apartment building.
HELD: YES.
Since petitioners opted to appropriate theimprovement for themselves as
early as June 1993,when they applied for a writ of execution
despiteknowledge that the auction sale did not include theapartment
building, they could not benefit from thelots improvement, until they
reimbursed the improver in full, based on the current market value of
theproperty.
Under Article 448, the landowner is giventhe option, either to appropriate
the improvement ashis own upon payment of the proper amount of
indemnity or to sell the land to the possessor in goodfaith.
Relatedly, Article 546 provides that a builder ingood faith is entitled to full
reimbursement for all thenecessary and useful expenses incurred; it also
giveshim right of retention until full reimbursement ismade.
The right of retention is considered as oneof the measures devised by the
law for the protectionof builders in good faith.
Its object is to guarantee fulland prompt reimbursement as it permits the
actualpossessor to remain in possession while he has notbeen reimbursed
(by the person who defeated him inthe case for possession of the property)
for thosenecessary expenses and useful improvements madeby him on the
thing possessed.
Given the circumstances of the instant casewhere the builder in good faith
has been clearlydenied his right of retention for almost half a decade,we
find that the increased award of rentals by theRTC was reasonable and
equitable.
NUGUID V CA
FACTS:
The deceased spouses Victorino and Crisanta dela Rosa were the registered
owners of a parcel of land situated in Bataan, and covered by OCT.
Victorino dela Rosa (widowed by then) sold one half of the said property
to Juliana Salazar for P 95.00. This sale was not registered.
Immediately after the sale, Juliana Salazar constructed a house on the lot
she purchased. Petitioner spouses caused the registration of a document
entitled Kasulatan ng Partihan at Bilihan.
In this document, Marciana dela Rosa, Victoria Buenaventura, Ernesto
Buenaventura, Virgilio Buenaventura, and Felicisimo Buenaventura-all heirs
of Victorino and Crisanta dela Rosa- sold to the petitioners the entire area
of the property for the sum of P300.00. S
ubsequently, the OCT was cancelled by the Register of Deeds, and TCT was
issued in the names of the petitioners.
The private respondents claim that the document is a forged deed. T
he petitioners assert that the land subject of this case was offered to them
for sale by Nicolas dela Rosa who then claimed that he had already
purchased the shares of the heirs over the subject property as evidenced by
a private document entitled Kasunduan.
The RTC dismissed the complaint filed by the private respondents, but on
appeal, this was reversed by the Court of Appeals.
Hence, this petition.
ISSUE: Whether or not the subsequent sale is valid, the petitioner spouses being
purchasers in good faith.
HELD:Yes.
The Original Certificate of Title No. 3778 covering the entire property was
clean and free from any annotation of an encumbrance, and there was
nothing whatsoever to indicate on its face any vice or infirmity in the title of
the registered owners-the spouses Victorino and Crisanta dela Rosa.
Thus, the petitioners could not have known of the prior sale to Juliana
Salazar as, precjsely, it was not registered.
The general rule is that if the property sold is registered land, the purchaser
in good faith has a right to rely on the certificate of title and is under no
duty to go behind it to look for flaws.
This notwithstanding, the petitioners did not rely solely upon the certificate
of title.
They personally inspected the subject property.
Undeniably, they found the same to be occupied by two houses, one
belonging to a certain Doray dela Rosa and the other to spouses Pedro
Guevarra and Pascuala Tolentino, parents of the respondents Guevarras.
Upon being informed of the petitioners desire to purchase the land, Doray
dela Rosa apparently offered to sell her house, which offer was accepted by
the petitioners.
As regards the spouses Guevarra, we find no reason to disturb the trial
courts finding that they themselves requested that they be allowed to
refrain on the property until such time that the petitioners would need the
entire premises; and in lieu of rentals to the petitioners, they offered to
continue paying the real estate taxes for one-half of the property as this
was their arrangement with the previous owners-to which request the
petitioners acceded.
Evidently, neither Doray dela Rosa nor the spouses Guevarra professed
ownership over the portions of land they were occupying; on the contrary,
by their actuations they expressly acknowledged that they were not the real
owners of the said property.
The spouses Guevarra, in particular, made no mention of the prior
unregistered sale to their predecessor-in-interest, Juliana Salazar.
Thus, when the petitioners registered the sale in their favor with the
Register of Deeds, they did so without any knowledge about the prior sale
in favor of Juliana Salazar.
The petitioners, therefore, had acted in good faith.
The petitionershad reaped all the benefits from the improvementintroduced
by the respondent during said period,without paying any amount to the
latter asreimbursement for his construction costs andexpenses.
They should account and pay for suchbenefits.
J.M. TUASON and CO., INC., petitioner, vs.ESTRELLA VDA. DE LUMANLAN
and the COURT OF APPEALS (FIFTH DIVISION), respondents.
J. M. Tuason & Co., Inc. petitioned for a review by certiorari of the decision issued by
the Court of Appeals (Fifth Division) in its case CA-G.R. No. 27259-R, reversing the
judgment rendered by the Court of First Instance of Rizal (Civil Case No. Q-4243) that
ordered defendant (now respondent) Estrella Vda. de Lumanlan to vacate the lot
occupied by her in Sta. Mesa Heights Subdivision, barrio Tatalon, Quezon City, and to
remove therefrom the house and other structures constructed thereon, paying
P240.00 a month until restoration of the premises to plaintiff.
The facts are stated in the decision of the Court of Appeals (accepted by both parties)
in this wise:1wph1.t
. . . That in the complaint filed in this case by plaintiff, J. M. Tuason & Co., Inc.,
hereinafter called Tuason, on 30 April, 1969, the basis is that it being the registered
owner of the property known as Santa Mesa Heights Subdivision, situated at Barrio
North Tatalon, Quezon City, herein defendant sometime in April, 1949 unlawfully
entered into possession of 800 square meters, and therein constructed his house so
that plaintiff prayed for ejectment and damages for the occupancy; and defendant in
her answer set forth affirmative defense that on 12 March, 1949, she had bought the
property she was occupying from one Pedro Deudor, and that in a compromise
agreement between Pedro and Tuason on 16 March 1953, approved by the Court of
First Instance of Quezon City, she was one of the buyers therein recognized, so that
she asked that her rights be recognized and the complaint dismissed; but on the basis
of the evidence presented by both parties in the trial, Lower Court sustained plaintiff,
holding that Tuason being the registered owner, and the question being purely one of
possession, therefore, defendant's said evidence was "completely immaterial". . . .
(Page 2 of Decision, Annex "A" of Petition.)
Upon the facts thus stated, the Fifth Division of the Court of Appeals held that,
pursuant to this Supreme Court's ruling in Evangelista vs. Deudor, L-12826,
September 10, 1959, the Compromise Agreement (Exh. 2) between the petitioner
Tuason & Co. and the Deudors constituted a valid defense against the possessory
action filed by Tuason & Co.; that under paragraph 7 of said Compromise Agreement,
petitioner bound and committed itself to sell to respondent Lumanlan the lot occupied
by her at a reasonable price; that said respondent had a right to compel petitioner to
accept payment for the lot in question; and that the compromise agreement legalized
the possession of respondent.
These pronouncements are assailed by the petitioner in this appeal as legally incorrect
and contrary to the decisions of this Court.
The terms of the compromise agreement between the heirs of Telesforo Deudor and
J. M. Tuason & Co. have been taken cognizance of in many decisions of this Court
(Evangelista vs. Deudor, jam. cit; Deudor vs. J. M. Tuason & Co., L-18768, May 30,
1961, and L-20105, Oct. 31, 1963; J. M. Tuason vs. Jaramillo, et al., L-18932-34,
Sept. 30, 1963; J. M. Tuason vs. Macalindong, L-15398, Dec. 29, 1962 and others).
The Deudors had therein recognized the registered title of Tuason & Co. over the
lands claimed by them, and received payment of certain sums of money; but as the
Deudors had, prior to the compromise, sold their possessory rights to various persons,
paragraph seventh of the compromise agreement (case Q-135 of the court of origin)
provided:
That the sales of the possessory rights claimed by the DEUDORS, are described in the
lists submitted by them to the OWNERS which are attached hereto marked Annexes
"B" and "C" and made part hereof. Whatever amounts may have been collected by
the DEUDORS on account thereof, shall be deducted from the total sum of
P1,201,063.00 to be paid to them. It shall be the joint and solidary obligation of the
DEUDORS to make the buyer of the lots purportedly sold by them to recognize the
title of the OWNERS over the property purportedly bought by them, and to make
them sign, whenever possible, new contracts of purchase for said property at the
current paces and terms specified by the OWNERS in their sales of lots in their
subdivision known at "Sta. Mesa Heights Subdivision." The DEUDORS HEREBY advised
the OWNERS that the buyer listed in Annex "B" herein with the annotation "continue"
shall buy the lots respectively occupied by them and shall sign contracts, but the sums
already paid by them to the DEUDORS amounting to P134,922.84 (subject to
verification by the Court) shall be credited to the buyers and shall be deducted from
the sums to be paid to the DEUDORS by the OWNERS. The DEUDORS also advise the
OWNERS that, the buyers listed in Annex "C" herein with the annotation "Refund"
have decided not to continue with their former contracts or purchases with the
DEUDORS and the sums already paid by them to the DEUDORS TOTALLING
P101,182.42 (subject to verification by the Court) shall be refunded to them by the
OWNERS and deducted from the sums that may be due to the DEUDORS from the
OWNERS (J.M. Tuason & Co., Inc. vs. Jaramillo, L-18932, Sept. 30, 1963);
Careful analysis of this paragraph of the compromise agreement will show that while
the same created "a sort of contractual relation" between the J. M. Tuason & Co.,
Inc., and the Deudor vendees (as ruled by this Court in Evangelista vs. Deudor, ante),
the same in no way obligated Tuason & Co. to sell to those buyers the lots occupied
by them at the price stipulated with the Deudors, but at "the current prices and terms
specified by the OWNERS (Tuason) in their sales of lots in their subdivision known as
'Sta. Mesa Heights Subdivision'". This is what is expressly provided. Further, the
paragraph plainly imports that these buyers of the Deudors must "recognize the title
of the OWNERS (Tuason) over the property purportedly bought by them" from the
Deudors, and "sign, whenever possible, new contracts of purchase for said property";
and, if and when they do so, "the sums paid by them to the Deudors . . . shall be
credited to the buyers." All that Tuason & Co. agreed to, therefore, was to grant the
Deudor buyers preferential right to purchase "at current prices and terms" the lots
occupied by them, upon their recognizing the title of Tuason & Co., Inc., and signing
new contracts therefor; and to credit them for the amounts they had paid to the
Deudors.
Nowhere in her answer did the respondent Estrella Vda. de Lumanlan claim that she
had signed a new contract with J. M. Tuason & Co., Inc. for the purchase of the lot
occupied. What is worse, instead of recognizing the title of the owners (Tuason & Co.)
as required by the aforementioned compromise agreement, she charged in paragraph
6 of her special defense (Rec. on Appeal, p. 10) that "Pedro Deudor and his co-
owners and the plaintiff herein . . . conspired together and helped each other . . . by
entering into a supposed Compromise" whereby "Pedro Deudor and his co-owners
renounced, ceded, waived and quitclaimed all their rights, title and interest in the
property including the land sold to herein defendant, in favor of the plaintiff J. M.
Tuason & Co., Inc., in consideration of the sum of P1,201,063.00, without the
knowledge and consent, and much less the intervention of the herein defendant." In
other words, the respondent Lumanlan in her answer repudiated and assailed the
compromise between the Deudors and J. M. Tuason & Co. How then can she now
claim to take advantage and derive rights from that compromise?
Without the compromise agreement, Lumanlan must justify her possession on the
basis of a pretended superiority of the Deudors' old Spanish informacion posesoria
over Tuason's Certificate of Title No. 1267, traceable back to the original Certificate of
Title No. 735 of Rizal, issued under the Registration Act No. 496. But, as ruled by this
Court in previous cases, Lumanlan is by now barred from assailing the decree of
registration in favor of Tuason & Co., Inc.'s predecessors twenty years after its
issuance (Tiburcio vs. PHHC, L-13429, Oct. 31, 1959; Tuason & Co. vs. Bolaos, 95
Phil. 107; Tuason & Co. vs. Santiago, 99 Phil. 622-623; Tuason & Co. vs.
Macalindong, supra; Tuason & Co. vs. Jaramillo, L-16827, Jan. 31, 1963).
It is thus apparent that no legal basis exists for the pronouncement in the appealed
decision that Tuason & Co. had committed itself to sell to Lumanlan the lot occupied
by her at a reasonable price, or that the compromise agreement legalized the
possession of the respondent, since the latter does not rely on the compromise but,
on the contrary, she assails it.
The Court of Appeals ruled that the price to be paid by Lumanlan to Tuason & Co.,
Inc., is governed by Article 1474 of the new Civil Code of the Philippines, which
provides that:
Where the price cannot be determined in accordance with the preceding articles, or in
any other manner, the contract is inefficacious. However, if the thing or any part
thereof has been delivered to and appropriated by the buyer, he must pay a
reasonable price therefor. What is a reasonable price is a question of fact dependent
on the circumstances of each particular case.
Since there has been no contract between petitioner Tuason & Co. and respondent
Lumanlan for the sale of the lot occupied by the latter, and by paragraph 7 of the
Compromise Agreement (assuming that respondent-appellee still has the right to
invoke the same, and seek refuge thereunder), Tuason & Co. did not consider itself
bound by the sales made by the Deudors, but demanded that the Deudor buyers
should sign new contracts with it at current prices specified for the sales of lots in
"Sta. Mesa Heights Subdivision" (ante) the aforequoted Article 1474 can have no
bearing on the case, Lumanlan not being a buyer from Tuason & Co.
As to Lumanlan's allegation in her counterclaim that she should be deemed a builder
in good faith, a similar contention has been rejected in Tuason & Co. vs. Macalindong,
L-15398, December 29, 1962, where we ruled that there being a presumptive
knowledge of the Torrens titles issued to Tuason & Co. and its predecessors-in-
interest since 1914, the buyer from the Deudors (or from their transferees) can not, in
good conscience, say now that she believed her vendor had rights of ownership over
the lot purchased. The reason given by the Court is that
Had he investigated before buying and before building his house on the questioned
lot, he would have been informed that the land is registered under the Torrens
system in the name of J. M. Tuason & Co., Inc., If he failed to make the necessary
inquiry, appellant is now bound conclusively by appellee's Torrens title (Sec. 51, Act
496; Emas vs. Zuzuarregui, 35 Phil. 144) (Tuason & Co., Inc. vs. Macalindong, ante).
Lumanlan had chosen to ignore the Torrens title of Tuason & Co., Inc. and relied
instead upon the Deudors' claim of ownership, perhaps because such course appeared
to her as more advantageous; hence, she has only herself to blame for the
consequences now that the Deudors' claim has been abandoned by the Deudors
themselves, and can not pretend good faith. The Court of First Instance, therefore,
did not err in holding that she was not a rightful possessor and sentencing her to
vacate.

Respondent could have asked that she recover or be credited with the amounts paid
by her to the Deudors, but as no claim to such credit was ever advanced by her in the
trial Court, no pronouncement can be made thereon in this appeal. Equity demands,
however, that her right to claim such return, or to have the amount offset against the
sums she was sentenced to pay, should be, as it is, reserved.
WHEREFORE, the decision of the Court of Appeals is reversed and that of the Court of
First Instance reinstated. Costs against respondent, Estrella Vda. de Lumanlan.
Technogas Philippines Manufacturing Corporation v. Court of Appeals
FACTS:
Technogas purchased a parcel of land from Pariz Industries, Inc. In the same year,
Eduardo Uy purchased the land adjacent to it. The following year, Uy bought another
lot adjoining the lot of Technogas. Portions of the buildings and wall bought by
Technogas together with the land from Pariz Industries are occupying a portion of
Uys adjoining land. The knowledge of some encroachment was only made known to
both parties after their parties of their respective parcels of land.
ISSUES:
1. Whether or not petitioner Technogas Philippines is a possessor in bad faith.
2. Whether or not petitioner Technogas Philippines has stepped into the shoes
of the seller.
HELD:
1.) No.
Unless one is versed in the science of surveying, no one can determine the
precise extent or location of his property by merely examining his paper
title.
There is no question in that when Technogas purchased the land from Pariz
Industries, the buildings and other structures were already in existence.
Furthermore, it is not clear as to who actually built these structures but it
can be assumed that the predecessor-in-interest of Technogas, Pariz
Industries, did so.
An article 527 of the New Civil Code presumes good faith. Since no proof
exists to show that the builder built the encroaching structures in bad faith,
the structures should be presumed to have been built in good faith.
Good faith consists in the belief of the builder that the land he is building on
is his, and his ignorance of any defect or flaw in his title.
Furthermore, possession acquired in good faith does not lose this character
except in case and from the moment facts exist which show that the
possessor is not aware that he possesses the thing improperly or
wrongfully.
The good faith ceases from the moment the defects in the title are made
known to the possessor, by extraneous evidence or by suit for recovery of
the property of the true owner.
2.) Yes.
Has been shown, contrary as to the good faith of Technogas has not
been overthrown.
Similarly, upon delivery of the property to Pariz Industries, as seller, to
Technogas, as buyer, the latter acquired ownership of the property.
Consequently, Technogas is deemed to have stepped into the shoes of
the seller with regard to all the rights of ownership of the property
over the immovable sold, including the right to compel Uy to exercise
either of the two options under Article 448 of the New Civil Code.
Thus, the landowners exercise of his option can only take place after
the builder shall have to know the intrusion in short, when both
parties shall have become aware of it.
Only then will the occasion for exercising the option arise, for it is only
then that both parties will have been aware that a problem exists with
regard to their property rights
Pleasantville Development Corporation v. Court of Appeals
FACTS:
On March 26, 1974, Wilson Kee on installment Lot 8 from C.T. Torres
Enterprises Inc. the exclusive real estate agent of petitioner.
Under the Contract to Sell on installment. Kee can exercise possession over
the parcel of land even before the completion of installment payments.
On January 20, 1975, Kee paid CTTEI relocation fee of Php 50.00 and
another on January 27, 1975 for the preparation of lot plan.
These amounts were paid by Kee before he took possession of Lot 8. After
the preparation of the lot plan and a copy was presented to Kee, Zenaida
Octaviano, employee of CTTEI accompanied Donnabelle Kee the wife of
Wilson Kee to inspect Lot 8.
Unfortuantely, Octaviano pointed Lot 9. Thereafter, Kee constructed his
residence on the said Lot 9 together a store, repair shop and other
improvements.
Edith Robillo purchased from Pleasantville Development Corporation Lot 9.
Sometime in 1975, she sold the said parcel of land, Lot 9, to Eldred
Jardinico which at that time is vacant.
Upon paying completely to Robillo, Jardinico secured from the Register of
Deeds of Bacolod City on December 19, 1978 Transfer Certificate of Title
No. 106367 in his name.
It was only that time that he discovered that Wilson Kee take possession of
that lot and that the same have introduced improvements to the same lot.
Jardinico confronted Kee and tried to reach for an amicable settlement, but
failed.
On January 30, 1981, Jardinico, through his lawyer, demanded that Kee
vacate Lot 9 and remove all the improvements introduced by the latter. Kee
refused which made Jardinico filed with the Municipal Trial Court in Cities,
Branch 3, Bacolod City a complaint for ejectment with damages against
Kee. Kee, in turn filed a third-party complaint against Pleasantville
Development Corporation and CTTEI.
The MTCC held that the erroneous delivery was attributable to CTTEI and
the Kee has no rights to Lot 9 because of the rescission made by CTTEI of
their contract due to Kees failure to pay the installment. MTCC also held
that Kee must pay reasonable rental for the use of Lot 9 and furthermore
he cannot claim reimbursement for the improvements introduced by him.
On appeal, the Regional Trial Court held that Pleasantville and CTTEI were
not negligent and that Kee was in bad faith.
Kee appealed directly to the Supreme Court which referred the matter to
the Court of Appeals.
The Appellate Court overturned the ruling of the RTC and held the Kee was
a builder in good faith and the erroneous delivery was attributable to the
negligence of CTTEI.
Hence the instant petition filed by Pleasantville.
ISSUES:
1. Whether or not, Wilson Kee is a builder in good faith.
2. Whether or not petitioner is liable for the acts of its agent CTTEI.
HELD:
1.
Petitioner fails to persuade the Court to abandon the findings and
conclusions of the Court of Appeals that Kee was a builder in good faith.
Good faith consists in the belief of the builder that the land he is building on
is his and his ignorance of any defect or flaw in his title.
And as good faith is presumed, petitioner has the burden of proving bad
faith on the part of Kee.
At the time he built improvements on Lot 8, Kee believed that said lot was
what he bought from petitioner.
He was not aware that the lot delivered to him was not Lot 8.
Thus, Kee is in good faith. Petitioner failed to prove otherwise.
To demonstrate Kee's bad faith, petitioner points to Kee's violation of
paragraphs 22 and 26 of the Contract of Sale on Installment. It has no
merit.
Such violations have no bearing whatsoever on whether Kee was a builder
in good faith, that is, on his state of mind at the time he built the
improvements on Lot 9.
These alleged violations may give rise to petitioner's cause of action against
Kee under the said contract (contractual breach), but may not be the basis
to negate the presumption that Kee was a builder in good faith.
2. Yes.
The rule is that the principal is responsible for the acts of the agent done
within the scope of his authority, and should bear the damage caused to
third persons. On the other hand, the agent who exceeds his authority is
personally liable for the damage.
But CTTEI was acting within its authority as the sole real estate
representative of petitioner when it made the delivery to Kee, only that in
so acting, it was negligent.
It is this negligence that is the basis of petitioner's liability, as principal of
CTTEI, per Articles 1909 and 1910 of the Civil Code.
For such negligence, the petitioner should be held liable for damages.
The rights of Kee and Jardinico vis-a-vis each other, as builder in good faith
and owner in good faith, respectively, are regulated by law (i.e., Arts. 448,
546 and 548 of the Civil Code).
It was error for the Court of Appeals to make a "slight modification" in the
application of such law [by holding petitioner and CTTEI solidarily liable], on
the ground of "equity".

You might also like